Đến nội dung

hoanglong2k nội dung

Có 992 mục bởi hoanglong2k (Tìm giới hạn từ 06-06-2020)



Sắp theo                Sắp xếp  

#555822 $ \frac{1}{(1-x)(1-y)(1-z)}+\frac{1}{(1+x)(1+y)(1+z)}\geq...

Đã gửi bởi hoanglong2k on 23-04-2015 - 13:28 trong Bất đẳng thức và cực trị

Ta có:

$-1 <x,y,z<1$

Do đó: $(1-x),(1-y),(1-z),(x+1),(y+1),(z+1)$ đều là các số dương.

Áp dụng $AM-GM$ ta được:

$LHS \geq \frac{1}{(\frac{1-x+1-y+1-z}{3})^3}+\frac{1}{(\frac{x+1+y+1+z+1}{3})^3}$

$=\frac{27}{(3-x-y-z)^3}+\frac{27}{(x+y+z+3)^3}$

$=(\frac{27}{(3-x-y-z)^3}+\frac{3-x-y-z}{3}+\frac{3-x-y-z}{3}+\frac{3-x-y-z}{3})+(\frac{27}{(x+y+z+3)^3}+\frac{x+y+z+3}{3}+\frac{x+y+z+3}{3}+\frac{x+y+z+3}{3})-6$

$\geq 4+4-6=2$

Xảy ra đẳng thức khi $x=y=z=0$

Khủng bố con họ à :D

Đặt $t=x+y+z$ thì $LHS\geq 27.\left [ \frac{1}{(3-t)^3}+\frac{1}{(3+t)^3} \right ]\geq 54.\sqrt{\frac{1}{[(3-t)(3+t)]^3}}\geq 54\sqrt{\frac{1}{9^3}}=2$




#555869 $ \frac{1}{(1-x)(1-y)(1-z)}+\frac{1}{(1+x)(1+y)(1+z)}\geq...

Đã gửi bởi hoanglong2k on 23-04-2015 - 19:50 trong Bất đẳng thức và cực trị

Thêm một cách khủng bổ nữa :D

$LHS=[\frac{1}{(1-x)(1-y)(1-z)}+(1-x)+(1-y)+(1-z)]+[\frac{1}{(x+1)(y+1)(z+1)}+(x+1)(y+1)(z+1)]-6$

$\geq 4+4-6=2$

Oh yeah, Another solution :v :D

We have : 

  $LHS\geq \frac{2}{\sqrt{(1-x^2)(1-y^2)(1-z^2)}}\geq \frac{2}{\sqrt{1.1.1}}=2$




#565241 $ \sum \frac{1}{a^3+b^3+1} \le 1$

Đã gửi bởi hoanglong2k on 12-06-2015 - 18:16 trong Bất đẳng thức và cực trị

Cho $abc=1$ ; $a,b,c>0$ . C/m $ \sum \frac{1}{a^3+b^3+1} \le 1$ 

  $\sum \frac{1}{a^3+b^3+1}\leq \sum \frac{1}{ab(a+b)+1}=\sum \frac{c}{a+b+c}=1$




#576726 $ (\sum x^{3})^{2} + 3(xyz)^{2}...

Đã gửi bởi hoanglong2k on 30-07-2015 - 11:23 trong Bất đẳng thức - Cực trị

Bài 2. Đầu tiên ta chứng minh: Với mọi số thực $a,b,c,d$ thỏa mãn $a+b+c+d=0$ thì $7(a^2+b^2+c^2+d^2)^2\geqslant 12(a^4+b^4+c^4+d^4)$

Thật vậy: $a^4+b^4+c^4=(a^2+b^2+c^2)^2-2(a^2b^2+b^2c^2+c^2a^2)\leqslant (a^2+b^2+c^2)^2-\dfrac{2(ab+bc+ca)^2}{3}$

Do đó ta chỉ cần chứng minh: $a^2+b^2+c^2+3(ab+bc+ca)\geqslant 0$

Mà $\sum\left(a^2+b^2+c^2+3ab+3bc+3ca\right)=0$ nên ta có thể giả sử $a^2+b^2+c^2+3(ab+bc+ca)\geqslant 0$

Áp dụng bất đẳng thức Cauchy-Schwarz: $(a+b)^4+(b+c)^4+(c+a)^4\geqslant \dfrac{(a^2+b^2+c^2+(a+b+c)^2)^2}{3}\geqslant \dfrac{4}{7}(a^4+b^4+c^4+(a+b+c)^4)$$\geqslant \dfrac{4}{7}(a^4+b^4+c^4)$

 Chỗ này có thể dùng BW để làm ( Trâu lắm =.= )

 Đặt $a+b=x;b+c=y;c+a=z$ thì $a=\frac{x+z-y}{2};b=\frac{x+y-z}{2};c=\frac{y+z-x}{2}$

 Ta chứng minh $(a+b)^4+(b+c)^4+(c+a)^4\geq\frac{4}{7}\left [a^4+b^4+c^4+(a+b+c)^4\right ]$

                         $\Leftrightarrow x^4+y^4+z^4\geq x^2y^2+y^2z^2+z^2x^2$ luôn đúng




#537903 $ (1+\frac{1}{n})^{n} < \fra...

Đã gửi bởi hoanglong2k on 14-12-2014 - 19:00 trong Bất đẳng thức và cực trị

Chứng minh rằng với mọi số nguyên  $n \geq 1$ ta có $ (1+\frac{1}{n})^{n} < \frac{11}{4}$

Bạn c/m bằng quy nạp ý :D




#560621 $(\frac{a}{a+b})^2 + (\frac{b}...

Đã gửi bởi hoanglong2k on 20-05-2015 - 23:01 trong Bất đẳng thức và cực trị

Cho a, b, c dương. Chứng minh rằng:

$(\frac{a}{a+b})^2 + (\frac{b}{b+c})^2 +(\frac{c}{a+c})^2 \geq \frac{3}{4}$

 

  BĐT $<=>\sum \left ( \frac{1}{1+\frac{b}{a}} \right )^2\geq \frac{3}{4}$

 Đặt $(x,y,z)=\left ( \frac{b}{a},\frac{c}{b},\frac{a}{c} \right )$ thì $xyz=1$

 Ta cần chứng minh $\sum \frac{1}{(1+x)^2}\geq \frac{3}{4}$

 Ta có : $\frac{1}{(1+x)^2}+\frac{1}{(1+y)^2}\geq \frac{1}{1+xy}=\frac{z}{z+1}$

 Nên ta chỉ cần chứng minh $\frac{z}{z+1}+\frac{1}{(z+1)^2}\geq \frac{3}{4}<=>(z-1)^2\geq 0$ ( đúng )

 Chứng minh xong

 Dấu "=" xảy ra khi $x=y=z=1$ hay $a=b=c$




#558330 $(\sqrt{a}+\sqrt{b})^{8}\ge...

Đã gửi bởi hoanglong2k on 08-05-2015 - 12:46 trong Bất đẳng thức và cực trị

1,Cho a,b>0. Chứng minh : $(\sqrt{a}+\sqrt{b})^{8}\geq 64ab(a+b)^{2}$

2, Cho x,y >0. Chứng minh: $\left ( \frac{x}{y} +\frac{y}{x}\right )^{32}\geq 8^{9}\left ( \frac{x^{2}}{y^{2}} +\frac{y^{2}}{x^{2}}\right )^{4}.\left ( \frac{x^{4}}{y^{4}} +\frac{y^{4}}{x^{4}}\right )$

1.       Ta có : 

  $$(\sqrt{a}+\sqrt{b})^8=(a+b+2\sqrt{ab})^4\geq \left [ 2\sqrt{(a+b).2\sqrt{ab}} \right ]^4=64ab(a+b)^2$$

2.       Áp dụng câu kết quả câu 1 ta có :

$$\left ( \frac{x}{y}+\frac{y}{x} \right )^8\geq 64\left ( \frac{x^2}{y^2}+\frac{y^2}{x^2} \right )^2$$

$$\Rightarrow \left ( \frac{x}{y}+\frac{y}{x} \right )^{32}\geq 8^8\left ( \frac{x^2}{y^2}+\frac{y^2}{x^2} \right )^8$$

          Lại có

 $$\left ( \frac{x^2}{y^2}+\frac{y^2}{x^2} \right )^4\geq \sqrt{64\left ( \frac{x^4}{y^4}+\frac{y^4}{x^4} \right )^2}=8\left ( \frac{x^4}{y^4}+\frac{y^4}{x^4} \right )$$

$$\Rightarrow  \left ( \frac{x}{y}+\frac{y}{x} \right )^{32}\geq 8^8\left ( \frac{x^2}{y^2}+\frac{y^2}{x^2} \right )^4.8\left ( \frac{x^4}{y^4}+\frac{y^4}{x^4} \right )= 8^9\left ( \frac{x^{2}}{y^{2}} +\frac{y^{2}}{x^{2}}\right )^{4}\left ( \frac{x^{4}}{y^{4}} +\frac{y^{4}}{x^{4}}\right )$$

 




#537699 $(1+x)(1+\frac{y}{x})(1+\frac{9}...

Đã gửi bởi hoanglong2k on 13-12-2014 - 17:49 trong Bất đẳng thức và cực trị

:D




#537518 $(1+x)(1+\frac{y}{x})(1+\frac{9}...

Đã gửi bởi hoanglong2k on 12-12-2014 - 21:17 trong Bất đẳng thức và cực trị

Đề phải thế này chứ: $(1+x)(1+\dfrac{y}{x})(1+\dfrac{9}{\sqrt{y}})^2 \ge 256$

Theo bunhia ta có:

$(1+x)(1+\dfrac{y}{x}) (1+\dfrac{9}{\sqrt{y}})^2 \ge (1+\sqrt{y})^2 (1+\dfrac{9}{\sqrt{y}})^2 \ge (1+\sqrt{y})^2(1+\dfrac{9}{\sqrt{y}})^2 \ge 256$

Cho em xin spam tí: Em biến đổi mãi mà cứ mắc $16(1+\sqrt{y})$ mãi, hóa ra đề sai ak ==''




#544486 $(a+b)(b+c)(c+a)\geq (ab+c)(bc+a)(ca+b)$

Đã gửi bởi hoanglong2k on 16-02-2015 - 16:14 trong Bất đẳng thức và cực trị

Cho a,b,c là các số thực dương có tổng bằng 3. Chứng minh rằng:

          $(a+b)(b+c)(c+a)\geq (ab+c)(bc+a)(ca+b)$




#531101 $(a+b)(b+c)(c+a)\geq 2(1+a+b+c)$

Đã gửi bởi hoanglong2k on 29-10-2014 - 20:03 trong Bất đẳng thức và cực trị

5. Cho $a;b;c>0$ . Chứng minh rằng $\frac{a}{a+b}+\frac{b}{b+c}+\frac{c}{c+a}< \sqrt{\frac{a}{b+c}}+\sqrt{\frac{b}{c+a}}+\sqrt{\frac{c}{a+b}}$

 

Áp dụng bất đẳng thức AM-GM ta có $\sqrt{\frac{a}{b+c}}=\frac{a}{\sqrt{a(b+c)}}\geq \frac{2a}{a+b+c}\Rightarrow \sum \sqrt{\frac{a}{b+c}}\geq 2$

Dấu "=" xảy ra khi $\left\{\begin{matrix} a=b+c\\ b=a+c\\ c=a+b \end{matrix}\right.\Leftrightarrow a=b=c=0$ (vô lí)

Vậy $\sum \sqrt{\frac{a}{b+c}}> 2$ (1)

Lại có $\frac{a}{a+b}-\frac{a+c}{a+b+c}=\frac{-ac}{(a+b)(a+b+c)}<0\Rightarrow \frac{a}{a+b}<\frac{a+c}{a+b+c}\Rightarrow \sum \frac{a}{a+b}<\sum \frac{a+c}{a+b+c}=2$ (2)

Từ (1) và (2) suy ra Đpcm :D




#594051 $(a+b+c)^{5}$$\geq$81($a^{2...

Đã gửi bởi hoanglong2k on 17-10-2015 - 13:55 trong Bất đẳng thức - Cực trị

Cho ba số dương a,b,c.CMR $(a+b+c)^{5}$$\geq$81($a^{2}$+$b^{2}$+$c^{2}$)abc.

 

 Không mất tính tổng quát, giả sử $a\leq b\leq c$

 Đặt $b=a+x$ và $c=a+x+y$ với $x,y\geq 0$

 Khi đó bất đẳng thức tương đương $(2x+y)^5+27a^3(x^2+xy+y^2)+9a^2(2x+y)^3+3a(26x^4+52x^3y+39x^2y^2+13xy^3+5y^4)\geq 0$

 Luôn đúng :v

 Đùa thôi :)) Cách trên trâu bò quá :))

 Lời giải :

 Áp dụng BĐT Cauchy-Schwarz ta có :

 $(ab+bc+ca)^2\geq 3abc(a+b+c)\Rightarrow 3abc\leq \frac{(ab+bc+ca)^2}{a+b+c}$

 Nên ta chỉ cần chứng minh

 $(a+b+c)^6\geq 27(a^2+b^2+c^2)(ab+bc+ca)^2$

 Để í : $(a+b+c)^6=[a^2+b^2+c^2+(ab+bc+ca)+(ab+bc+ca)]^3$

 Áp dụng AM-GM ta có ngay điều cần chứng minh

 Dấu bằng xảy ra khi $a=b=c$




#609811 $(a^{2}+b^{2}+abc)(b^{2}+c^{2}+a...

Đã gửi bởi hoanglong2k on 19-01-2016 - 15:27 trong Bất đẳng thức và cực trị

Câu $iii)$

Áp dụng BĐT AM-GM thì cần chứng minh: $(a+b)(b+c)(c+a)\geq (c+ab)(a+bc)(b+ca)$

Ta có: $(c+ab)(a+bc)\leq \frac{[c+a+b(c+a)]^2}{4}=\frac{(c+a)^2(b+1)^2}{4}$

Tương tự 2 BĐT kia rồi nhân lại AM-GM phát nữa là ra.

P/s: Thầy có ra bài iii) đâu nhỉ :D

Câu $ii)$ còn một khoảng "rất nhỏ" giữa $1$ với $1,05$ mà không ra, ức chế thật -_-

 Câu iii) UCT cho lẹ :D

 

$ii)$ Cho $a;b;c>0$ và $a+b+c=3$. Chứng minh rằng

$\frac{a}{3a^{2}+abc+27}+\frac{b}{3b^{2}+abc+27}+\frac{c}{3c^{2}+abc+27} \leq \frac{3}{31}$

 Ta có : BĐT$\Leftrightarrow \sum \left (\dfrac{a}{27}-\dfrac{a}{3a^2+abc+27}\right )\geq \dfrac{12}{31}$

                    $\Leftrightarrow \sum \dfrac{3a^3+a^2bc}{3a^2+abc+27}\geq \dfrac{12}{31}$

 Áp dụng BĐT Cauchy-Schwarz ta có :

$$\sum \dfrac{3a^3}{3a^2+a^2bc+27}\geq \dfrac{3(a^2+b^2+c^2)^2}{3(a^3+b^3+c^3)+3abc+81}$$

$$abc\sum \dfrac{a}{3a^2+abc+27}\geq \dfrac{9abc}{3(a^3+b^3+c^3)+3abc+81}$$

 Nên ta chỉ cần chứng minh $\dfrac{3(a^2+b^2+c^2)^2+9abc}{3(a^3+b^3+c^3)+3abc+81}\geq \dfrac{12}{31}$

Hay $93(a^2+b^2+c^2)^2+279abc\geq 36(a^3+b^3+c^3)+36abc+972$

       $\Leftrightarrow 93(a^2+b^2+c^2)^2+243abc\geq 36(a^3+b^3+c^3)+972$

 Đặt $p=a+b+c=3;q=ab+bc+ca;r=abc$ thì ta cần chứng minh

       $93(9-2q)^2+243r\geq 36(27-9q+3r)+972$

       $\Leftrightarrow 124q^2-1008q+1863+45r\geq 0$

 Áp dụng BĐT Schur ta có $r\geq \dfrac{4q-9}{3}$ nên ta chỉ cần chứng minh

       $124q^2-1008q+1863+15(4q-9)\geq 0\Leftrightarrow (q-3)(31q-144)\geq 0$

 Luôn đúng vì $q\leq 3$

 Vậy BĐT được chứng minh xong. Dấu "=" xảy ra khi $a=b=c=1$




#610442 $(a^{2}+b^{2}+abc)(b^{2}+c^{2}+a...

Đã gửi bởi hoanglong2k on 22-01-2016 - 22:34 trong Bất đẳng thức và cực trị

iii) UCT không được, tính đạo hàm cấp 2 ra $\leq 0$ nhé!

 OK, nhầm :lol:

 

$i)$ Cho các số dương $a;b;c$ thỏa mãn $a^{2}+b^{2}+c^{2} \geq 3$
Chứng minh rằng $(a^{2}+b^{2}+abc)(b^{2}+c^{2}+abc)(c^{2}+a^{2}+abc) \geq 3abc(a+b+c)^{2}$

 Theo nguyên lí Dirichlet, giả sử $(a^2-abc)(b^2-abc)\geq 0\Leftrightarrow a^2b^2+a^2b^2c^2\geq a^3bc+ab^3c$

 Do đó mà $(b^2+c^2+abc)(c^2+a^2+abc)\geq (a^2+b^2+c^2)(c^2+2abc)$

 Nên ta chỉ cần chứng minh $(a^2+b^2+c^2)(c^2+2abc)(a^2+b^2+abc)\geq 3abc(a+b+c)^2$

 Áp dụng BĐT Cauchy-Schwarz ta có $(c^2+abc+abc)(abc+a^2+b^2)\geq abc(a+b+c)^2$ và $a^2+b^2+c^2\geq 3$ theo giả thiết

 Từ đó suy ra điều phải chứng minh

 Dấu "=" xảy ra khi và chỉ khi $a=b=c=1$




#575753 $(a^2-a+1)(b^2-b+1)(c^2-c+1) \ge 1$

Đã gửi bởi hoanglong2k on 27-07-2015 - 00:28 trong Bất đẳng thức và cực trị

cho $a,b,c \ge 0$ thỏa mãn $a+b+c=3$. cmr $(a^2-a+1)(b^2-b+1)(c^2-c+1) \ge 1$

 Theo nguyên lí Dirichlet, giả sử $(a-1)(b-1)\geq 0$

 Khi đó : $(a^2-a+1)(b^2-b+1)=ab(a-1)(b-1)+a^2+b^2-a-b+1\geq \frac{1}{2}(a+b)^2-(a+b)+1=\frac{1}{2}(3-c)^2-(3-c)+1=\frac{1}{2}(c^2-4c+5)$

 Nên ta chỉ cần chứng minh $(c^2-c+1)(c^2-4c+5)\geq 2\Leftrightarrow (c-1)^2(c^2-3c+3)\geq 0$ đúng

 Từ đó suy ra $(a^2-a+1)(b^2-b+1)(c^2-c+1) \geq 1$




#577988 $(a^{2}b+b^{2}c+c^{2}a)(ab^{2}+bc^{2}+ca^{2}+abc)\leq 8$

Đã gửi bởi hoanglong2k on 02-08-2015 - 22:03 trong Bất đẳng thức và cực trị

Cho $a;b;c>=0 ; a+2b+3c=4$.Chứng minh rằng:

$(a^{2}b+b^{2}c+c^{2}a)(ab^{2}+bc^{2}+ca^{2}+abc)\leq 8$

 Áp dụng bất đẳng thức AM-GM ta có:

 $\frac{1}{2}.(a^2b+b^2c+c^2a).2(ab^2+bc^2+ca^2+abc)\leq \frac{\left[a^2b+b^2c+c^2a+2(ab^2+bc^2+ca^2)+2abc\right]^2}{8}$

 Lại có:

  $a^2b+b^2c+c^2a+2(ab^2+bc^2+ca^2)+2abc$

  $\leq a^2b+b^2c+c^2a+2(ab^2+bc^2+ca^2)+\frac{9}{2}abc$

  $=\frac{(a+2b)(b+2c)(c+2a)}{2} =\frac{(a+2b)(4b+8c)(c+2a)}{8}$

  $\leq \frac{\left(\frac{a+2b+4b+8c+c+2a}{3}\right)^3}{8}$ 

  $=\frac{(a+2b+3c)^3}{8}=8$

  Dấu "=" xảy ra khi $a=2;b=1;c=0$




#570017 $(ab)^{3}+(bc)^{3}+(ca)^{3}\leq...

Đã gửi bởi hoanglong2k on 05-07-2015 - 12:20 trong Bất đẳng thức và cực trị

Ta có: $VT=(\sum ab)(\sum (ab)^2-abc \sum a)=q(q^2-3r)$

Theo Schur $p^3+9r \geq 4pq=> r \geq \frac{4q-1}{9}$

Thế vào trên ta được $q(q^2-\frac{3(4q-1)}{9}) \leq \frac{1}{64}$ với $q \leq \frac{1}{3}$ hy vọng đúng

 Anh xem lại khai triển ạ 

 

 

Cho 3 số thực không âm a,b,c thỏa mãn $a+b+c=1$. Chứng minh rằng:

 $(ab)^{3}+(bc)^{3}+(ca)^{3}\leq \frac{1}{64}$

 Viết lại bất đẳng thức bằng ngôn ngữ $p,q,r$ thì ta cần chứng minh 

   $q^3-3qr+3r^2\leq \frac{1}{64}$

 Với $q\leq \frac{1}{4}$ thì ta có :

   $q\geq 9r\geq r\Rightarrow q^3-3qr+3r^2\leq \frac{1}{64}+3r(r-q)\leq \frac{1}{64}$

 Với $\frac{1}{3}\geq q\geq \frac{1}{4}$, áp dụng Schur ta có :

   $q\leq \frac{9r+1}{4}\Rightarrow q^3-3qr+3r^2\leq \left ( \frac{9r+1}{4} \right )^3-\frac{3r}{4}+3r^2$

 Ta cần chứng minh $\left ( \frac{9r+1}{4} \right )^3-\frac{3r}{4}+3r^2\leq \frac{1}{64}$ với $r\in \left [ 0;\frac{1}{27} \right ]$

 Biến đổi tương đương là ra

   




#571648 $(b+c)^4+(a+c)^4+(a+b)^4\geq \frac{4}{7}(a...

Đã gửi bởi hoanglong2k on 12-07-2015 - 09:26 trong Bất đẳng thức và cực trị

Với các số thực a,b,c.Chứng minh rằng

$(b+c)^4+(a+c)^4+(a+b)^4\geq \frac{4}{7}(a^4+b^4+c^4) $

 Do $\sum a(a+b+c)=(\sum a)^2\geq 0$ nên 1 trong 3 số $a(a+b+c)~;b(a+b+c)~;c(a+b+c)$ không âm

 Không mất tính tổng quát, giả sử $a(a+b+c)\geq 0$

 Khi đó :

 Đặt $f(a,b,c)=(b+c)^4+(a+c)^4+(a+b)^4- \frac{4}{7}(a^4+b^4+c^4)$ 

$f(a,b,c)-f(a,\frac{b+c}{2},\frac{b+c}{2})=\left [ \frac{3(b^2+c^2)}{28}+3a(a+b+c)+\frac{15(b+c)^2}{56} \right ](b-c)^2\geq 0$

 $\Rightarrow f(a,b,c)\geq f(a,\frac{b+c}{2},\frac{b+c}{2})$

 Khúc còn lại đạo hàm là ra




#552320 $(O;R)$ đường kính $AB$...$AP$là tiếp tuyến tại...

Đã gửi bởi hoanglong2k on 08-04-2015 - 01:59 trong Hình học

Untitledbcd6c.png

 Gọi D là giao điểm HK và BP

Ta có: DK//PA ( cùng vuông góc AB )

      $\Rightarrow \frac{DK}{PA}=\frac{BK}{AB}=\frac{BK}{2R}$                               (1)

 Xét các $\Delta KBH$ và $\Delta AOP$ vuông có $\widehat{AOP}=\widehat{KBH}$ ( đồng vị, OP//MB do cùng vuông góc AM )

  $\Rightarrow \Delta KBH\sim \Delta AOP\Rightarrow \frac{KH}{AP}=\frac{KB}{OA}=\frac{KB}{R}$ (2)

Từ (1) và (2) $\Rightarrow DK=\frac{1}{2}.KH$ nên D là trung điểm KH 




#530240 [TOPIC] Luyện tập biến đổi căn thức

Đã gửi bởi hoanglong2k on 23-10-2014 - 22:00 trong Đại số

giúp em bài toán khó này với ạ Thank các bác

bạn up đề cho rõ đi nhé




#530286 [TOPIC] Luyện tập biến đổi căn thức

Đã gửi bởi hoanglong2k on 24-10-2014 - 00:01 trong Đại số

 

3/ Tính : $E= x^{3}+y^{3} -3(x+y)+2014$
Với $x=\sqrt[3]{3+2\sqrt{2}} +\sqrt[3]{3-2\sqrt{2}}$ 

      $y=\sqrt[3]{17+12\sqrt{2}} +\sqrt[3]{17-12\sqrt{2}}$
 

bạn tính $x^{3},y^{3}$ và đưa nó về 1 pt bậc ẩn x,y rồi tính theo đó




#537153 [Toán 9]Tuyển tập bổ đề hình học

Đã gửi bởi hoanglong2k on 11-12-2014 - 00:30 trong Hình học

Bài toán phụ 1: Cho (O) và A nằm ngoài (O). Kẻ tiếp tuyến AB,AC với B,C là các tiếp điểm. Đường thẳng qua A cắt (O) tại D,E. AO cắt BC tại H. Chứng minh rằng D,E,H,O cùng nằm trên một đường tròn.

 

 

 

https://www.facebook...&type=1

Ta có $\Delta ABD\sim \Delta AEB\Rightarrow AB^2=AE.AD$

Mà theo hệ thức lượng: $AB^2=AO.AH\Rightarrow AE.AD=AO.AH\Rightarrow \Delta ADH\sim\Delta AOE\Rightarrow \widehat{AHD}=\widehat{AEO}\Rightarrow$ Tứ giác OHDE nội tiếp




#617007 [Trường Xuân toán học miền nam 2016] Vietnam TST 2016 MOCK Test 2

Đã gửi bởi hoanglong2k on 26-02-2016 - 13:22 trong Thi HSG cấp Tỉnh, Thành phố. Olympic 30-4. Đề thi và kiểm tra đội tuyển các cấp.

 Bài 5. 

 Gọi $A_3$ là giao điểm $AA_1$ với $(O)$

 Khi đó $AA_2$ là đường đối trung của tam giác $ABC$ nên $\Delta ABA_2\sim \Delta AA_3C\Rightarrow AA_2.AA_3=AB.AC$

 Mặt khác sử dụng đẳng thức Ptolemy ta có $A_1A.A_1A_3=A_1B.A_1C\Leftrightarrow AA_1(AA_3-AA_1)=\dfrac{BC^2}{4}\Rightarrow AA_1.AA_3=\dfrac{AB^2+AC^2}{2}$

 Từ đó suy ra $\dfrac{AA_2}{AA_1}=\dfrac{2AB.AC}{AB^2+AC^2}$

 Đặt $AB=c;BC=a;CA=b$ thì ta có $\dfrac{AA_2}{AA_1}=\dfrac{2bc}{b^2+c^2}$, tương tư cho các phân thức kia

 Mà theo công thức tính diện tích tam giác ta lại có $S=\dfrac{abc}{4R}=pr$

$\Rightarrow S^2=\dfrac{abcpr}{4R}\Rightarrow \dfrac{4r}{R}=\dfrac{16S^2}{abcp}=\dfrac{16p(p-a)(p-b)(p-c)}{abcp}=\dfrac{2(a+b-c)(b+c-a)(c+a-b)}{abc}$

 Vì thế nên ta chỉ cần chứng minh $\dfrac{2bc}{b^2+c^2}+\dfrac{2ca}{c^2+a^2}+\dfrac{2ab}{a^2+b^2}\geq 1+\dfrac{2(a+b-c)(b+c-a)(c+a-b)}{abc}$

 Bất đẳng thức tương đương với :

$\sum (b-c)^2\left ( \dfrac{b+c-a}{abc}-\dfrac{1}{b^2+c^2} \right )\geq 0\Leftrightarrow S_a(b-c)^2+S_b(c-a)^2+S_c(a-b)^2\geq 0$ trong đó $S_a=\dfrac{b+c-a}{abc}-\dfrac{1}{b^2+c^2}$, ...

 Giả sử $a\geq b\geq c$, khi đó dễ dàng chứng minh được $S_b\geq 0$ và $S_b+S_c\geq 0$

 Mặt khác, do $a,b,c$ là 3 cạnh của một tam giác nên ta cũng chứng minh $S_a+S_b\geq 0$

 Khi đó theo tiêu chuẩn SOS ta có điều cần chứng minh

 Dấu "=" xảy ra khi và chỉ khi $a=b=c$ hay $\Delta ABC$ là tam giác đều :)




#616742 [Trường Xuân toán miền Nam]Vietnam TST 2016 Mock Test 1

Đã gửi bởi hoanglong2k on 24-02-2016 - 20:01 trong Thi HSG cấp Tỉnh, Thành phố. Olympic 30-4. Đề thi và kiểm tra đội tuyển các cấp.

Mình xin làm bài bất đẳng thức

Đặt $a+b+c=p , ab+bc+ca=q , abc=r $

Ta cần chứng minh bđt 

$\frac{q+\sqrt{q^2-2pr}}{p+\sqrt{p^2-2q}} \geq \frac{p}{3} $

Bằng cách bình phương 2 vế, ta đưa được về dạng $f(r) \geq 0$

Mà ta có $f'(r) \geq 0 $ nên theo định lý $PQR$, ta chỉ cần chứng minh bđt trong trường hợp có $2$ số bằng nhau 

Không mất tính tổng quát, giả sử $a=b $

Chuẩn hóa $abc=1 => c=\frac{1}{a^2} $

BĐT trở thành

$f(a)=a^2-\frac{2}{a} +\frac{1}{a^4} + 2\sqrt{2a^4+1} +\sqrt{\frac{2}{a^2}+ \frac{1}{a^6}}-3\sqrt{a^4+\frac{2}{a^2}} \geq 0 $

Khảo sát hàm 1 biến, ta thấy $f(a) \geq f(1) = 0$

Do đó, ta có điều phải chứng minh

Dấu bằng xảy ra khi $a=b=c $

Cách này hơi trâu

Mong chờ một lời giải đẹp từ các bạn 

 Bài này cho đơn giản thì chuẩn hóa $p=3$ sau đó dùng bất đẳng thức Schur $r\geq \dfrac{4q-9}{3}$ sẽ đơn giản hơn nhiều 

 Ta có bất đẳng thức tương đương $\dfrac{q+\sqrt{q^2-2pr}}{p+\sqrt{p^2-2q}}\leq \dfrac{p}{3}$

 Chuẩn hóa $p=3$ thì cần chứng minh $q+\sqrt{q^2-6r}\leq 3+\sqrt{9-2q}$

 Áp dụng bất đẳng thức Schur thì $r\geq \dfrac{4q-9}{3}$, ta cần chứng minh $q+\sqrt{q^2-8q+18}\leq 3+\sqrt{9-2q}$

 Đây là bất đẳng thức một biến, xét đạo hàm sẽ ra. Hoặc biến đổi tương đương cho đỡ ( chúa ghét đạo hàm chứa căn -_- )

 Bất đẳng thức cần chứng minh tương đương với $(3-q).\left (1-\dfrac{3-q}{\sqrt{q^2-8q+18}+\sqrt{9-2q}}\right )\geq 0$

 Luôn đúng do $3\geq q$ và $\sqrt{q^2-8q+18}>\sqrt{q^2-6q+9}\geq 3-q$

 Dấu "=" khi $a=b=c$




#614592 [Đại số]THPT tháng 12: $a^2+b^2+c^2-ab-bc-ca \geqslant k\left|...

Đã gửi bởi hoanglong2k on 12-02-2016 - 21:25 trong Thảo luận đề thi VMEO IV

BĐT này tương đương với:

$2(a^2+b^2+c^2-ab-bc-ca)\geq\frac{k(a-b)(a-c)(b-c)(ab+bc+ca)}{(a+b)(b+c)(c+a)}$ nên nó gần như giống với đề tháng 12 này, chỉ khác ở chỗ một bên là $k$ với dấu giá trị tuyệt đối, một bên là $\frac{k}{2}$.

 Thực ra thì bỏ dấu giá trị tuyệt đối vào kết quả cho $k$ cũng không khác nhau là mấy vì khi giảm $a,b,c$ đi cùng một lượng thì các đại lượng trên cũng không đổi, và ý tưởng của bài này cũng là đưa một biến về $0$ để xét hàm còn lại 2 biến